Does the series \sum^{\infty}_{n=1}sin(\frac{1}{n^{4}}) converge?

Click For Summary
The series ∑sin(1/n^4) is being analyzed for convergence. The nth term test indicates the limit of the sequence approaches zero, suggesting it does not diverge. The integral test is not applicable due to the non-monotonic nature of the sequence, while the root test is deemed ineffective. A limit comparison test with 1/n^4 is proposed as a potential method for determining convergence. Ultimately, the discussion emphasizes the importance of using appropriate tests for series convergence.
Bipolarity
Messages
773
Reaction score
2

Homework Statement



Determine whether the following series diverges, converges conditionally, or converges absolutely.

\sum^{\infty}_{n=1}sin(\frac{1}{n^{4}})

Homework Equations


The Attempt at a Solution


This was on today's test, and was the only problem I wasn't able to solve. I doubt my teacher will be going over these, and in any case his explanations never satisfy me, so could someone help me with this?

According to the nth term test, the limit of the sequence is 0, since sin(x) is continuous, so the function doesn't necessarily diverge.

Integral test can't be applied because the sequence is not monotonic. Root test serves no purpose. Limit comparison might work, but with what? Ratio does not work (I think?).

How might I approach this?

BiP
 
Physics news on Phys.org
Hedi beat me to it down below VVV haha. I forgot that something bigger that diverges tells you nothing.
 
Last edited:
use limit comparison with 1/n^4.for large n this is a positive series.
 
hedipaldi said:
use limit comparison with 1/n^4.for large n this is a positive series.

Thanks!

But damn! I wish I thought of that in the test! Oh well.

BiP
 
Question: A clock's minute hand has length 4 and its hour hand has length 3. What is the distance between the tips at the moment when it is increasing most rapidly?(Putnam Exam Question) Answer: Making assumption that both the hands moves at constant angular velocities, the answer is ## \sqrt{7} .## But don't you think this assumption is somewhat doubtful and wrong?

Similar threads

  • · Replies 3 ·
Replies
3
Views
1K
  • · Replies 7 ·
Replies
7
Views
2K
  • · Replies 2 ·
Replies
2
Views
2K
  • · Replies 5 ·
Replies
5
Views
2K
  • · Replies 1 ·
Replies
1
Views
2K
  • · Replies 1 ·
Replies
1
Views
1K
  • · Replies 4 ·
Replies
4
Views
2K
  • · Replies 3 ·
Replies
3
Views
2K
  • · Replies 17 ·
Replies
17
Views
2K
  • · Replies 4 ·
Replies
4
Views
2K